Difference between revisions of "G285 2021 MC10B"

(Created page with "==Problem 1== Find <math>\left \lceil {\frac{3!+4!+5!+6!}{2+3+4+5+6}} \right \rceil</math> <math>\textbf{(A)}\ \frac{290}{7}\qquad\textbf{(B)}\ \frac{890}{21}\qquad\textbf{(C...")
 
m (Problem 1)
Line 2: Line 2:
 
Find <math>\left \lceil {\frac{3!+4!+5!+6!}{2+3+4+5+6}} \right \rceil</math>
 
Find <math>\left \lceil {\frac{3!+4!+5!+6!}{2+3+4+5+6}} \right \rceil</math>
  
<math>\textbf{(A)}\ \frac{290}{7}\qquad\textbf{(B)}\ \frac{890}{21}\qquad\textbf{(C)}\ \frac{89}{2}\qquad\textbf{(D)}\ \frac{87}{2}\qquad\textbf{(E)}\ \frac{223}{5}</math>
+
<math>\textbf{(A)}\ 42\qquad\textbf{(B)}\ 43\qquad\textbf{(C)}\ 44\qquad\textbf{(D)}\ 45\qquad\textbf{(E)}\ 46</math>
  
 
[[G285 MC10B Problems/Problem 1|Solution]]
 
[[G285 MC10B Problems/Problem 1|Solution]]

Revision as of 16:52, 12 May 2021

Problem 1

Find $\left \lceil {\frac{3!+4!+5!+6!}{2+3+4+5+6}} \right \rceil$

$\textbf{(A)}\ 42\qquad\textbf{(B)}\ 43\qquad\textbf{(C)}\ 44\qquad\textbf{(D)}\ 45\qquad\textbf{(E)}\ 46$

Solution

Problem 2

If $deg(Q(x))=3$, and $deg(K(x))=2$, and $Q(x)=(x-2)K(x)$, what is $deg(Q(x)-2K(x))$?

$\textbf{(A)}\ 0\qquad\textbf{(B)}\ 1\qquad\textbf{(C)}\ 2\qquad\textbf{(D)}\ 3\qquad\textbf{(E)}\ 4$

Solution

Problem 3

A convex hexagon of length $s$ is inscribed in a circle of radius $r$, where $r \neq s$. If $\frac{s}{2r}=\frac{21}{29}$, and $rs=58$, find the area of the hexagon.

$\textbf{(A)}\ 60\qquad\textbf{(B)}\ 90\qquad\textbf{(C)}\ 120\qquad\textbf{(D)}\ 240\qquad\textbf{(E)}\ 480$

Solution